How many ways to fill in a square grid with certain restrictions












4














Suppose I have a 5x5 grid of squares. I would like to fill in 15 checkmarks in the squares such that (1) each of the 25 square cells contains at most one checkmark, (2) each row has exactly 3 checkmarks, and (3) each column has exactly 3 checkmarks. How many ways are there to fill in these 15 checkmarks?



More generally, suppose I have an $n times n $ square grid, and I would like to fill in $mn$ checkmarks such that (1) each of the $n^2$ square cells contains at most one checkmark, (2) each row has exactly m checkmarks, and (3) each column has exactly m checkmarks. How many ways are there to do so? If $m=1,$ I think the answer is $n!$. But I am not sure about the general case.



Also, if I have an additional restriction that no checkmarks on the diagonal, i.e., no checkmark in the (1,1), (2,2),... (n,n) cells. How many ways are there?



Thanks very much! Wish all very happy new year!










share|cite|improve this question






















  • If you remove the only one checkmark per box requirement then you are looking at the Ehrhart polynomial of the Birkhoff polytope, and get into the Anand-Dumir-Gupta conjecture and related areas.
    – Sam Hopkins
    10 hours ago
















4














Suppose I have a 5x5 grid of squares. I would like to fill in 15 checkmarks in the squares such that (1) each of the 25 square cells contains at most one checkmark, (2) each row has exactly 3 checkmarks, and (3) each column has exactly 3 checkmarks. How many ways are there to fill in these 15 checkmarks?



More generally, suppose I have an $n times n $ square grid, and I would like to fill in $mn$ checkmarks such that (1) each of the $n^2$ square cells contains at most one checkmark, (2) each row has exactly m checkmarks, and (3) each column has exactly m checkmarks. How many ways are there to do so? If $m=1,$ I think the answer is $n!$. But I am not sure about the general case.



Also, if I have an additional restriction that no checkmarks on the diagonal, i.e., no checkmark in the (1,1), (2,2),... (n,n) cells. How many ways are there?



Thanks very much! Wish all very happy new year!










share|cite|improve this question






















  • If you remove the only one checkmark per box requirement then you are looking at the Ehrhart polynomial of the Birkhoff polytope, and get into the Anand-Dumir-Gupta conjecture and related areas.
    – Sam Hopkins
    10 hours ago














4












4








4







Suppose I have a 5x5 grid of squares. I would like to fill in 15 checkmarks in the squares such that (1) each of the 25 square cells contains at most one checkmark, (2) each row has exactly 3 checkmarks, and (3) each column has exactly 3 checkmarks. How many ways are there to fill in these 15 checkmarks?



More generally, suppose I have an $n times n $ square grid, and I would like to fill in $mn$ checkmarks such that (1) each of the $n^2$ square cells contains at most one checkmark, (2) each row has exactly m checkmarks, and (3) each column has exactly m checkmarks. How many ways are there to do so? If $m=1,$ I think the answer is $n!$. But I am not sure about the general case.



Also, if I have an additional restriction that no checkmarks on the diagonal, i.e., no checkmark in the (1,1), (2,2),... (n,n) cells. How many ways are there?



Thanks very much! Wish all very happy new year!










share|cite|improve this question













Suppose I have a 5x5 grid of squares. I would like to fill in 15 checkmarks in the squares such that (1) each of the 25 square cells contains at most one checkmark, (2) each row has exactly 3 checkmarks, and (3) each column has exactly 3 checkmarks. How many ways are there to fill in these 15 checkmarks?



More generally, suppose I have an $n times n $ square grid, and I would like to fill in $mn$ checkmarks such that (1) each of the $n^2$ square cells contains at most one checkmark, (2) each row has exactly m checkmarks, and (3) each column has exactly m checkmarks. How many ways are there to do so? If $m=1,$ I think the answer is $n!$. But I am not sure about the general case.



Also, if I have an additional restriction that no checkmarks on the diagonal, i.e., no checkmark in the (1,1), (2,2),... (n,n) cells. How many ways are there?



Thanks very much! Wish all very happy new year!







co.combinatorics






share|cite|improve this question













share|cite|improve this question











share|cite|improve this question




share|cite|improve this question










asked 12 hours ago









KPU

563




563












  • If you remove the only one checkmark per box requirement then you are looking at the Ehrhart polynomial of the Birkhoff polytope, and get into the Anand-Dumir-Gupta conjecture and related areas.
    – Sam Hopkins
    10 hours ago


















  • If you remove the only one checkmark per box requirement then you are looking at the Ehrhart polynomial of the Birkhoff polytope, and get into the Anand-Dumir-Gupta conjecture and related areas.
    – Sam Hopkins
    10 hours ago
















If you remove the only one checkmark per box requirement then you are looking at the Ehrhart polynomial of the Birkhoff polytope, and get into the Anand-Dumir-Gupta conjecture and related areas.
– Sam Hopkins
10 hours ago




If you remove the only one checkmark per box requirement then you are looking at the Ehrhart polynomial of the Birkhoff polytope, and get into the Anand-Dumir-Gupta conjecture and related areas.
– Sam Hopkins
10 hours ago










2 Answers
2






active

oldest

votes


















5














What you are looking for is the number of matrices in the class $mathcal A(n,m)$ of $(0,1)$-matrices that are $ntimes n$ and have each row and column containing exactly $m$ entries equal to $1$. This is a pretty well-studied question, but an exact answer isn't known except in some small cases.



You are right that for $m=1$, the number is $n!$, as then you are counting the $ntimes n$ permutation matrices. A sort of generalization of this was obtained in




Wei, Wan-Di. "The class $mathfrak A(R,S)$ of $(0,1)$-matrices." Discrete Mathematics 39, no. 3 (1982): 301–305. Journal link




where it is given that the number of such matrices is at least



$$frac{(n!)^m}{(m!)^n}.$$



I say it is “sort of” a generalization because it is only a lower bound. Again, there is no closed form known for the exact number. I think the most frequently-cited asymptotic results are those of McKay and others, for example see




McKay, Brendan D., Wang, Xiaoji. "Asymptotic enumeration of 0-1 matrices with equal row sums and equal column sums." Linear Algebra and its Applications. 373 (2003): 273–287. Journal link




and anything that cites it.



There are other results – and other asymptotic results – out there. A good starting point for more details is the book Combinatorial Matrix Classes by Brualdi.



Finally, note that this is the same as counting balanced regular bipartite graphs. For example, your question is the same as this one where the $d_v$ and $d_c$ of that question are taken to be equal. So you may wish to see the answer – provided by McKay! – appearing there.






share|cite|improve this answer





















  • Thanks so much for the very helpful information!! Happy New Year!
    – KPU
    3 hours ago



















0














The problem amounts to counting binary or $0/1$-matrices with restrictions on row/column sums.



In general, let the row sums be $r(n)=(r_1,dots,r_n)$ and column sums $c(n)=(c_1,dots,c_n)$. Obviously, $0leq r_i, c_jleq n$ for all $i, j$. Further, let $mathbf{x}=(x_1,dots,x_n)$ and $mathbf{y}=(y_1,dots,y_n)$ with the multi-exponent notation $mathbf{x}^{u}=x_1^{u_1}cdots x_n^{u_n}$. Then, there is this generating function
$$prod_{i,j=1}^n(1+x_iy_j)=sum_{r(n),c(n)} N(r(n),c(n))mathbf{x}^{r(n)}mathbf{y}^{c(n)} tag1$$
for the enumeration $N(r(n),c(n))$ of the number of binary matrices with row sums $r(n)$ and $c(n)$.



To get back to your question, extract the coefficient $N((k,dots,k),(k,dots,k))$ of
$$mathbf{x}^{(k,dots,k)}mathbf{y}^{(k,dots,k)}$$
from the above product in (1).






share|cite|improve this answer























    Your Answer





    StackExchange.ifUsing("editor", function () {
    return StackExchange.using("mathjaxEditing", function () {
    StackExchange.MarkdownEditor.creationCallbacks.add(function (editor, postfix) {
    StackExchange.mathjaxEditing.prepareWmdForMathJax(editor, postfix, [["$", "$"], ["\\(","\\)"]]);
    });
    });
    }, "mathjax-editing");

    StackExchange.ready(function() {
    var channelOptions = {
    tags: "".split(" "),
    id: "504"
    };
    initTagRenderer("".split(" "), "".split(" "), channelOptions);

    StackExchange.using("externalEditor", function() {
    // Have to fire editor after snippets, if snippets enabled
    if (StackExchange.settings.snippets.snippetsEnabled) {
    StackExchange.using("snippets", function() {
    createEditor();
    });
    }
    else {
    createEditor();
    }
    });

    function createEditor() {
    StackExchange.prepareEditor({
    heartbeatType: 'answer',
    autoActivateHeartbeat: false,
    convertImagesToLinks: true,
    noModals: true,
    showLowRepImageUploadWarning: true,
    reputationToPostImages: 10,
    bindNavPrevention: true,
    postfix: "",
    imageUploader: {
    brandingHtml: "Powered by u003ca class="icon-imgur-white" href="https://imgur.com/"u003eu003c/au003e",
    contentPolicyHtml: "User contributions licensed under u003ca href="https://creativecommons.org/licenses/by-sa/3.0/"u003ecc by-sa 3.0 with attribution requiredu003c/au003e u003ca href="https://stackoverflow.com/legal/content-policy"u003e(content policy)u003c/au003e",
    allowUrls: true
    },
    noCode: true, onDemand: true,
    discardSelector: ".discard-answer"
    ,immediatelyShowMarkdownHelp:true
    });


    }
    });














    draft saved

    draft discarded


















    StackExchange.ready(
    function () {
    StackExchange.openid.initPostLogin('.new-post-login', 'https%3a%2f%2fmathoverflow.net%2fquestions%2f319842%2fhow-many-ways-to-fill-in-a-square-grid-with-certain-restrictions%23new-answer', 'question_page');
    }
    );

    Post as a guest















    Required, but never shown

























    2 Answers
    2






    active

    oldest

    votes








    2 Answers
    2






    active

    oldest

    votes









    active

    oldest

    votes






    active

    oldest

    votes









    5














    What you are looking for is the number of matrices in the class $mathcal A(n,m)$ of $(0,1)$-matrices that are $ntimes n$ and have each row and column containing exactly $m$ entries equal to $1$. This is a pretty well-studied question, but an exact answer isn't known except in some small cases.



    You are right that for $m=1$, the number is $n!$, as then you are counting the $ntimes n$ permutation matrices. A sort of generalization of this was obtained in




    Wei, Wan-Di. "The class $mathfrak A(R,S)$ of $(0,1)$-matrices." Discrete Mathematics 39, no. 3 (1982): 301–305. Journal link




    where it is given that the number of such matrices is at least



    $$frac{(n!)^m}{(m!)^n}.$$



    I say it is “sort of” a generalization because it is only a lower bound. Again, there is no closed form known for the exact number. I think the most frequently-cited asymptotic results are those of McKay and others, for example see




    McKay, Brendan D., Wang, Xiaoji. "Asymptotic enumeration of 0-1 matrices with equal row sums and equal column sums." Linear Algebra and its Applications. 373 (2003): 273–287. Journal link




    and anything that cites it.



    There are other results – and other asymptotic results – out there. A good starting point for more details is the book Combinatorial Matrix Classes by Brualdi.



    Finally, note that this is the same as counting balanced regular bipartite graphs. For example, your question is the same as this one where the $d_v$ and $d_c$ of that question are taken to be equal. So you may wish to see the answer – provided by McKay! – appearing there.






    share|cite|improve this answer





















    • Thanks so much for the very helpful information!! Happy New Year!
      – KPU
      3 hours ago
















    5














    What you are looking for is the number of matrices in the class $mathcal A(n,m)$ of $(0,1)$-matrices that are $ntimes n$ and have each row and column containing exactly $m$ entries equal to $1$. This is a pretty well-studied question, but an exact answer isn't known except in some small cases.



    You are right that for $m=1$, the number is $n!$, as then you are counting the $ntimes n$ permutation matrices. A sort of generalization of this was obtained in




    Wei, Wan-Di. "The class $mathfrak A(R,S)$ of $(0,1)$-matrices." Discrete Mathematics 39, no. 3 (1982): 301–305. Journal link




    where it is given that the number of such matrices is at least



    $$frac{(n!)^m}{(m!)^n}.$$



    I say it is “sort of” a generalization because it is only a lower bound. Again, there is no closed form known for the exact number. I think the most frequently-cited asymptotic results are those of McKay and others, for example see




    McKay, Brendan D., Wang, Xiaoji. "Asymptotic enumeration of 0-1 matrices with equal row sums and equal column sums." Linear Algebra and its Applications. 373 (2003): 273–287. Journal link




    and anything that cites it.



    There are other results – and other asymptotic results – out there. A good starting point for more details is the book Combinatorial Matrix Classes by Brualdi.



    Finally, note that this is the same as counting balanced regular bipartite graphs. For example, your question is the same as this one where the $d_v$ and $d_c$ of that question are taken to be equal. So you may wish to see the answer – provided by McKay! – appearing there.






    share|cite|improve this answer





















    • Thanks so much for the very helpful information!! Happy New Year!
      – KPU
      3 hours ago














    5












    5








    5






    What you are looking for is the number of matrices in the class $mathcal A(n,m)$ of $(0,1)$-matrices that are $ntimes n$ and have each row and column containing exactly $m$ entries equal to $1$. This is a pretty well-studied question, but an exact answer isn't known except in some small cases.



    You are right that for $m=1$, the number is $n!$, as then you are counting the $ntimes n$ permutation matrices. A sort of generalization of this was obtained in




    Wei, Wan-Di. "The class $mathfrak A(R,S)$ of $(0,1)$-matrices." Discrete Mathematics 39, no. 3 (1982): 301–305. Journal link




    where it is given that the number of such matrices is at least



    $$frac{(n!)^m}{(m!)^n}.$$



    I say it is “sort of” a generalization because it is only a lower bound. Again, there is no closed form known for the exact number. I think the most frequently-cited asymptotic results are those of McKay and others, for example see




    McKay, Brendan D., Wang, Xiaoji. "Asymptotic enumeration of 0-1 matrices with equal row sums and equal column sums." Linear Algebra and its Applications. 373 (2003): 273–287. Journal link




    and anything that cites it.



    There are other results – and other asymptotic results – out there. A good starting point for more details is the book Combinatorial Matrix Classes by Brualdi.



    Finally, note that this is the same as counting balanced regular bipartite graphs. For example, your question is the same as this one where the $d_v$ and $d_c$ of that question are taken to be equal. So you may wish to see the answer – provided by McKay! – appearing there.






    share|cite|improve this answer












    What you are looking for is the number of matrices in the class $mathcal A(n,m)$ of $(0,1)$-matrices that are $ntimes n$ and have each row and column containing exactly $m$ entries equal to $1$. This is a pretty well-studied question, but an exact answer isn't known except in some small cases.



    You are right that for $m=1$, the number is $n!$, as then you are counting the $ntimes n$ permutation matrices. A sort of generalization of this was obtained in




    Wei, Wan-Di. "The class $mathfrak A(R,S)$ of $(0,1)$-matrices." Discrete Mathematics 39, no. 3 (1982): 301–305. Journal link




    where it is given that the number of such matrices is at least



    $$frac{(n!)^m}{(m!)^n}.$$



    I say it is “sort of” a generalization because it is only a lower bound. Again, there is no closed form known for the exact number. I think the most frequently-cited asymptotic results are those of McKay and others, for example see




    McKay, Brendan D., Wang, Xiaoji. "Asymptotic enumeration of 0-1 matrices with equal row sums and equal column sums." Linear Algebra and its Applications. 373 (2003): 273–287. Journal link




    and anything that cites it.



    There are other results – and other asymptotic results – out there. A good starting point for more details is the book Combinatorial Matrix Classes by Brualdi.



    Finally, note that this is the same as counting balanced regular bipartite graphs. For example, your question is the same as this one where the $d_v$ and $d_c$ of that question are taken to be equal. So you may wish to see the answer – provided by McKay! – appearing there.







    share|cite|improve this answer












    share|cite|improve this answer



    share|cite|improve this answer










    answered 10 hours ago









    Louis Deaett

    7551618




    7551618












    • Thanks so much for the very helpful information!! Happy New Year!
      – KPU
      3 hours ago


















    • Thanks so much for the very helpful information!! Happy New Year!
      – KPU
      3 hours ago
















    Thanks so much for the very helpful information!! Happy New Year!
    – KPU
    3 hours ago




    Thanks so much for the very helpful information!! Happy New Year!
    – KPU
    3 hours ago











    0














    The problem amounts to counting binary or $0/1$-matrices with restrictions on row/column sums.



    In general, let the row sums be $r(n)=(r_1,dots,r_n)$ and column sums $c(n)=(c_1,dots,c_n)$. Obviously, $0leq r_i, c_jleq n$ for all $i, j$. Further, let $mathbf{x}=(x_1,dots,x_n)$ and $mathbf{y}=(y_1,dots,y_n)$ with the multi-exponent notation $mathbf{x}^{u}=x_1^{u_1}cdots x_n^{u_n}$. Then, there is this generating function
    $$prod_{i,j=1}^n(1+x_iy_j)=sum_{r(n),c(n)} N(r(n),c(n))mathbf{x}^{r(n)}mathbf{y}^{c(n)} tag1$$
    for the enumeration $N(r(n),c(n))$ of the number of binary matrices with row sums $r(n)$ and $c(n)$.



    To get back to your question, extract the coefficient $N((k,dots,k),(k,dots,k))$ of
    $$mathbf{x}^{(k,dots,k)}mathbf{y}^{(k,dots,k)}$$
    from the above product in (1).






    share|cite|improve this answer




























      0














      The problem amounts to counting binary or $0/1$-matrices with restrictions on row/column sums.



      In general, let the row sums be $r(n)=(r_1,dots,r_n)$ and column sums $c(n)=(c_1,dots,c_n)$. Obviously, $0leq r_i, c_jleq n$ for all $i, j$. Further, let $mathbf{x}=(x_1,dots,x_n)$ and $mathbf{y}=(y_1,dots,y_n)$ with the multi-exponent notation $mathbf{x}^{u}=x_1^{u_1}cdots x_n^{u_n}$. Then, there is this generating function
      $$prod_{i,j=1}^n(1+x_iy_j)=sum_{r(n),c(n)} N(r(n),c(n))mathbf{x}^{r(n)}mathbf{y}^{c(n)} tag1$$
      for the enumeration $N(r(n),c(n))$ of the number of binary matrices with row sums $r(n)$ and $c(n)$.



      To get back to your question, extract the coefficient $N((k,dots,k),(k,dots,k))$ of
      $$mathbf{x}^{(k,dots,k)}mathbf{y}^{(k,dots,k)}$$
      from the above product in (1).






      share|cite|improve this answer


























        0












        0








        0






        The problem amounts to counting binary or $0/1$-matrices with restrictions on row/column sums.



        In general, let the row sums be $r(n)=(r_1,dots,r_n)$ and column sums $c(n)=(c_1,dots,c_n)$. Obviously, $0leq r_i, c_jleq n$ for all $i, j$. Further, let $mathbf{x}=(x_1,dots,x_n)$ and $mathbf{y}=(y_1,dots,y_n)$ with the multi-exponent notation $mathbf{x}^{u}=x_1^{u_1}cdots x_n^{u_n}$. Then, there is this generating function
        $$prod_{i,j=1}^n(1+x_iy_j)=sum_{r(n),c(n)} N(r(n),c(n))mathbf{x}^{r(n)}mathbf{y}^{c(n)} tag1$$
        for the enumeration $N(r(n),c(n))$ of the number of binary matrices with row sums $r(n)$ and $c(n)$.



        To get back to your question, extract the coefficient $N((k,dots,k),(k,dots,k))$ of
        $$mathbf{x}^{(k,dots,k)}mathbf{y}^{(k,dots,k)}$$
        from the above product in (1).






        share|cite|improve this answer














        The problem amounts to counting binary or $0/1$-matrices with restrictions on row/column sums.



        In general, let the row sums be $r(n)=(r_1,dots,r_n)$ and column sums $c(n)=(c_1,dots,c_n)$. Obviously, $0leq r_i, c_jleq n$ for all $i, j$. Further, let $mathbf{x}=(x_1,dots,x_n)$ and $mathbf{y}=(y_1,dots,y_n)$ with the multi-exponent notation $mathbf{x}^{u}=x_1^{u_1}cdots x_n^{u_n}$. Then, there is this generating function
        $$prod_{i,j=1}^n(1+x_iy_j)=sum_{r(n),c(n)} N(r(n),c(n))mathbf{x}^{r(n)}mathbf{y}^{c(n)} tag1$$
        for the enumeration $N(r(n),c(n))$ of the number of binary matrices with row sums $r(n)$ and $c(n)$.



        To get back to your question, extract the coefficient $N((k,dots,k),(k,dots,k))$ of
        $$mathbf{x}^{(k,dots,k)}mathbf{y}^{(k,dots,k)}$$
        from the above product in (1).







        share|cite|improve this answer














        share|cite|improve this answer



        share|cite|improve this answer








        edited 11 hours ago

























        answered 11 hours ago









        T. Amdeberhan

        17.1k228126




        17.1k228126






























            draft saved

            draft discarded




















































            Thanks for contributing an answer to MathOverflow!


            • Please be sure to answer the question. Provide details and share your research!

            But avoid



            • Asking for help, clarification, or responding to other answers.

            • Making statements based on opinion; back them up with references or personal experience.


            Use MathJax to format equations. MathJax reference.


            To learn more, see our tips on writing great answers.





            Some of your past answers have not been well-received, and you're in danger of being blocked from answering.


            Please pay close attention to the following guidance:


            • Please be sure to answer the question. Provide details and share your research!

            But avoid



            • Asking for help, clarification, or responding to other answers.

            • Making statements based on opinion; back them up with references or personal experience.


            To learn more, see our tips on writing great answers.




            draft saved


            draft discarded














            StackExchange.ready(
            function () {
            StackExchange.openid.initPostLogin('.new-post-login', 'https%3a%2f%2fmathoverflow.net%2fquestions%2f319842%2fhow-many-ways-to-fill-in-a-square-grid-with-certain-restrictions%23new-answer', 'question_page');
            }
            );

            Post as a guest















            Required, but never shown





















































            Required, but never shown














            Required, but never shown












            Required, but never shown







            Required, but never shown

































            Required, but never shown














            Required, but never shown












            Required, but never shown







            Required, but never shown







            Popular posts from this blog

            Quarter-circle Tiles

            build a pushdown automaton that recognizes the reverse language of a given pushdown automaton?

            Mont Emei